LSAT and Law School Admissions Forum

Get expert LSAT preparation and law school admissions advice from PowerScore Test Preparation.

 Administrator
PowerScore Staff
  • PowerScore Staff
  • Posts: 8916
  • Joined: Feb 02, 2011
|
#35171
Complete Question Explanation

Flaw in the Reasoning—CE. The correct answer choice is (E)

Your task in this Method of Reasoning question is to select the answer choice containing the most
accurate description of the flawed argument in the stimulus. The argument proceeds:

..... Premise: ..... our club recruited the best volleyball players in the city

..... Sub-Conclusion: ..... thus, we will have the best team in the city

..... Premise: ..... the best team in the city will be the team most likely to win the city
..... ..... ..... ..... championship

..... Conclusion: ..... so, our club will almost certainly be city champions this year

Your prephrase is that the argument contains two flaws. First, the subsidiary conclusion results from
an error of composition, inferring that the team as a whole will share a characteristic, being the best,
held by its individual team members. Next, the conclusion that the teams will almost certainly be city
champions improperly results from a relativity flaw. It is inappropriate to infer from the comparative
premise, that the team is the most likely of the teams to win, the non-comparative conclusion that the
team almost certainly will win. The correct answer choice will describe this flawed argumentation.

The incorrect answers will not describe these flaws. Instead they will describe argumentation that
did not occur in the stimulus, or they will accurately describe a portion of the argument that is not
flawed.

Answer choice (A): The argument did not presume the team, an entity, would be best solely on
the basis of competition. Rather, it stated the team will be best because the club recruited the best
volleyball players in the city.

Answer choice (B): The conclusion that the team would be successful, i.e., win the city
championship, resulted from the claim that the team will be the best in the city. While the argument
was flawed in other ways, the talent of the team is not irrelevant to the success of the team.

Answer choice (C): The argument predicted the outcome of the championship based on the premise
that the best team will be the team most likely to win the championship, and was not based solely on
a comparison between the parties in the competition.

Answer choice (D): This choice describes an error of division, the reverse of the error of composition
reasoning flaw that lead to the sub-conclusion.

Answer choice (E): This is the correct answer choice.While the stimulus contained two reasoning
flaws, the test makers chose to describe only one of them in this answer choice, the relativity flaw,
in which the argument inferred from the team being the one most likely to win that it would almost
certainly win.
 Sherry001
  • Posts: 81
  • Joined: Aug 18, 2014
|
#21827
Hello ,
I had trouble with this one , because I was convinced I had identified the flaw in the argument ! I thought the flaw to be "individual traits to conclude the best team" . I realize the author doesn't conclude about the best team , and I am not to attack a premise. BUT he uses it to support his intermediate conclusion. So why is this not a flaw ? :( NOT COOL !

1- we have recruited the best players in the city.
Inte con: we will have the best team in the city .

2- we are likely to win , because we are the best team.

Conclusion : our club will almost certainly be city champion this year.


My analysis: I see two issues here. First of all just because you have the best players does not mean they will make the best team. Maybe they'll hate each other. 2) um the author said likely ... But in the conclusion he seems pretty darn sure.


A) nope. Also that they were the best players. And never said only.
B) nope. predicts the success of the team not the individual . And comparison was relevant.
c) nope. Just like A
D) the only reason I avoided this was because I saw it as part to part or else part to whole.
E)yes sir !


Thank you so much
Sherry
 David Boyle
PowerScore Staff
  • PowerScore Staff
  • Posts: 836
  • Joined: Jun 07, 2013
|
#21870
Sherry001 wrote:Hello ,
I had trouble with this one , because I was convinced I had identified the flaw in the argument ! I thought the flaw to be "individual traits to conclude the best team" . I realize the author doesn't conclude about the best team , and I am not to attack a premise. BUT he uses it to support his intermediate conclusion. So why is this not a flaw ? :( NOT COOL !

1- we have recruited the best players in the city.
Inte con: we will have the best team in the city .

2- we are likely to win , because we are the best team.

Conclusion : our club will almost certainly be city champion this year.


My analysis: I see two issues here. First of all just because you have the best players does not mean they will make the best team. Maybe they'll hate each other. 2) um the author said likely ... But in the conclusion he seems pretty darn sure.


A) nope. Also that they were the best players. And never said only.
B) nope. predicts the success of the team not the individual . And comparison was relevant.
c) nope. Just like A
D) the only reason I avoided this was because I saw it as part to part or else part to whole.
E)yes sir !


Thank you so much
Sherry
Hello Sherry,

"First of all just because you have the best players does not mean they will make the best team. Maybe they'll hate each other." may be true, but answer D gets that part-to-whole thing backwards, discussing the logical flaw known as an error of division, instead of discussing an error of composition, which could describe one flaw in the stimulus.
So the best answer is E, one which addresses the other problem you saw, "2) um the author said likely ... But in the conclusion he seems pretty darn sure." Donald Trump may have, say, 35% of the primary vote, so has the "best" chance, but he does not have over 50%, which sort of equates to "more likely to occur than not" in the stimulus.

Hope this helps,
David
 karen4300
  • Posts: 9
  • Joined: Jun 28, 2018
|
#48241
For a.c D), when it says "that event is more likely to occur than not", means that the event will "almost certainly" occur? Idk how that describes the flaw because I don't find "more likely than not" to parallel absolute certainty like the flaw in the stimulus did?
 Adam Tyson
PowerScore Staff
  • PowerScore Staff
  • Posts: 5153
  • Joined: Apr 14, 2011
|
#49069
I assume you mean answer choice E, Karen? I see where you're going and why you're confused! "More likely" does not mean "almost certain", but that's not the flow of information in this stimulus. Rather, it's the other way around - if the author thinks that victory is "almost certain", that means he has to believe it is "more likely than not". We can't put a number on "almost certain" with any certainty, but let's say it's at least a 75% chance of winning. Less than that and I think "almost certain" is no longer appropriate (and I lean more towards something like 90+% before I would say "almost certain"). 75% more than meets the standard of "more likely than not, because that only requires that we be above 50%!

So, the author went from claiming that our team has the best chance (compared to all the other teams) to believing that our team will almost certainly win (way more likely than not). Answer E was putting it mildly, probably to make it less attractive to us, but it's still a valid description of what the author did.

The language in the answer doesn't need to parallel the language in the stimulus perfectly - that's a different question type. Instead, it just needs to accurately describe at least one error in the stimulus, which in this case had a combination of a part-to-whole flaw and a numbers and percentages flaw. Answer E describes the numbers and percentages flaw accurately, if somewhat meekly, and so it's good enough for our purposes.
 ericau02
  • Posts: 73
  • Joined: Feb 19, 2019
|
#64407
Hi can you please further explain what the Relativity flaw is? Thank you
 James Finch
PowerScore Staff
  • PowerScore Staff
  • Posts: 943
  • Joined: Sep 06, 2017
|
#64949
Hi Erica,

The relativity flaw is when we have a relative comparison that is then used to infer something absolute (or almost-absolute, in this case) or vice-versa. This is commonly tested on the LSAT; just because we know which of two things is more likely to happen, doesn't mean we know that either one is likely to happen. Similarly, just because we know something is likely to happen, doesn't necessarily mean we know that something else is less or more likely to happen (unless the two are mutually exclusive).

In this case, we know that the club in the stimulus is the "most likely" team to win the title, a relative comparison. But since we don't know what their actual odds are, or how many other teams there are and what those teams' odds are, then we can't infer whether this team is likely or not to actually win the title.

Hope this clears things up!
 180bound
  • Posts: 34
  • Joined: Jun 11, 2019
|
#76263
Hi, could someone explain to me why choice B is not consistent with the explanation of this problem: "First, the subsidiary conclusion results from an error of composition, inferring that the team as a whole will share a characteristic, being the best,
held by its individual team members"?
 Adam Tyson
PowerScore Staff
  • PowerScore Staff
  • Posts: 5153
  • Joined: Apr 14, 2011
|
#76897
The problem with answer B, 180bound, is that the quality of the players on the team may indeed be relevant to the team's success. The flaw is not that the quality of the players is not relevant, but rather that the quality of the players is not sufficient to prove the quality of the team. A factor can be relevant but still be insufficient!

A "part to whole" answer would have been great for this question, but answer B is not a part to whole answer. It's a "general lack of relevant evidence" answer, and that wasn't the problem in this argument. Answer B is a trap!
User avatar
 Albertlyu
  • Posts: 98
  • Joined: Jul 18, 2020
|
#82579
thank you, although, I am still confused about the relativity flaw:

Let us say there are 4 teams playing against each other, A, B, C, D. Team A's got a 35% chance of winning which is higher than everyone else, since there are only 4 teams, and A has got a bigger chance than everybody else, why can't we say A is most likely to be the winner? Please can anyone give me a counterexample or a scenario in which the conclusion is refuted? thanks.

Get the most out of your LSAT Prep Plus subscription.

Analyze and track your performance with our Testing and Analytics Package.